Вы находитесь на странице: 1из 65

Exam Title : TS01 polity test L1

Email : sandeepbiswas221@gmail.com
Contact : 8250416813

QUESTION 1.
Consider the following:

1. Governor

2. Chief minister

www.freeupscmaterials.org
3. Council of ministers

g
4. Advocate general

or
Which of the above is/are part of state executive?

s.
a) 1 and 2 only
b) 2 and 3 only
c) 1, 2 and 3 only
d) All of the above
al
ri
Correct Answer: D
Your Answer: C
te

Explanation

-The state executive consists of the governor, the chief minister, the council of ministers and the advocate
ma

general of the state.


sc

-The governor is the chief executive head of the state.


up

QUESTION 2.
Consider the following statements about Scheduled and Tribal Areas
ee

1. Governor is empowered to declare an area to be a scheduled area with the consent of president.
fr

2. Governor is empowered to direct that any laws/act of parliament and state legislature does not apply to
scheduled area or apply with some modifications and exceptions.

3. The jurisdiction of high court over suits and cases of tribal areas is specified by the president.

Which of the following statements is/are correct?

a) only 1,2 and 3


b) only 2 and 3
c) only 2
d) 1 and 2

Civilsdaily
Page 0
Email: hello@civilsdaily.com
Exam Title : TS01 polity test L1
Email : sandeepbiswas221@gmail.com
Contact : 8250416813

Correct Answer: C
Your Answer: Unanswered
Explanation

Q.2 ) - President is empowered to declare any area to be scheduled area.

- Governor has a special responsibility to submit a report annually or whenever so required by the president

www.freeupscmaterials.org
-The jurisdiction of high court over suits and cases of tribal areas is specified by the governor.

g
or
QUESTION 3.
Indian constitution is found on the bedrock of the balance between the fundamental rights and the directive

s.
principles. The goals set out by the directive principles have to be achieved without the abrogation of the
means provided by the fundamental rights. This statement was held by the Supreme Court in which famous
case?
al
a) Kesavananda Bharti case
ri
b) Minerva mills case
c) Golaknath case
te

d) Berburi union case


Correct Answer: B
ma

Your Answer: B
Explanation
sc

QUESTION 4.
Which of the following provisions of the Indian Constitution secures the term 'Justice' present in the
up

Preamble?

1. Fundamental Rights
ee

2. Directive Principles of the State Policy


fr

3. Judicial Review

Select the correct option using the codes given below.

a) 1, 2 and 3
b) 1 and 3 only
c) 1 and 2 only
d) 1 only
Correct Answer: C
Your Answer: C

Civilsdaily
Page 0
Email: hello@civilsdaily.com
Exam Title : TS01 polity test L1
Email : sandeepbiswas221@gmail.com
Contact : 8250416813

Explanation

The term justice in the Preamble embraces three distinct formssocial, economic and political, secured
through various provisions of Fundamental Rights and Directive Principles.

Social justice denotes the equal treatment of all citizens without any social distinction based on caste,
colour, race, religion, sex and so on. It means absence of privileges being extended to any particular

www.freeupscmaterials.org
section of the society, and improvement in the conditions of backward classes (SCs, STs and OBCs) and
women.

g
or
Economic justice denotes the non-discrimination between people on the basis of economic factors. It
involves the elimination of glaring inequalities in wealth, income and property. A combination of social

s.
justice and economic justice denotes what is known as distributive justice.

Political justice implies that all citizens should have equal political rights, equal access to all political offices
and equal voice in the government.
al
ri
The ideal of justicesocial, economic and politicalhas been taken from the Russian Revolution (1917).
te

QUESTION 5.
Consider the following regarding the office of Whip-
ma

1. The office of Whip is mentioned in the Parliamentary Statute.


sc

2. He is appointed by the Speaker.

Which of the statements given above is/are correct?


up

a) 1 only
ee

b) 2 only
c) Both 1 and 2
fr

d) Neither 1 nor 2
Correct Answer: D
Your Answer: D
Explanation

-The office of whip is mentioned neither in the Constitution of India nor in the Rules of the House nor in
a Parliamentary Statute. It is based on the conventions of the parliamentary government.

-Every political party, whether ruling or Opposition has its own whip in the Parliament. He is appointed by
the political party to serve as an assistant floor leader.

QUESTION 6.

Civilsdaily
Page 0
Email: hello@civilsdaily.com
Exam Title : TS01 polity test L1
Email : sandeepbiswas221@gmail.com
Contact : 8250416813

Consider the difference between adjournment and prorogation

1. Adjournment only terminates a sitting and not a session of the House while prorogation not only
terminates a sitting but also a session of the House.

2. Adjournment is done by presiding officer of the House while prorogation is done by the president of
India.

www.freeupscmaterials.org
Which of the statements given above is/are correct?

g
or
a) 1 only
b) 2 only
c) Both 1 and 2

s.
d) Neither 1 nor 2
Correct Answer: C
Your Answer: Unanswered
al
Explanation
ri

Both statements are correct.


te

QUESTION 7.
ma

Consider the following statement regarding National emergency

1. The six fundamental rights under Article 19 can be suspended when the National Emergency is declared
sc

on the ground of war, external aggression or armed rebellion


up

2. The resolution of disapproval of National emergency must be passed by Lok Sabha and Rajya Sabha
with simple majority.
ee

Which of the above statements are incorrect?


fr

a) 1 only
b) 2 only
c) Both 1 and 2
d) Neither 1 nor 2
Correct Answer: C
Your Answer: A
Explanation

1.The six fundamental rights under article 19 can be suspended only when the national emergency is
declared on the ground of war or external aggression and not on the ground of armed rebellion.

Civilsdaily
Page 0
Email: hello@civilsdaily.com
Exam Title : TS01 polity test L1
Email : sandeepbiswas221@gmail.com
Contact : 8250416813

2. The resolution of disapproval of National emergency must be passed by Lok Sabha only with a simple
majority.

QUESTION 8.
Consider the following statements:

1. He decides on the questions of the qualifications of the MP in consultation with the Election commission

www.freeupscmaterials.org
2. He can appoint a commission to investigate into the conditions of SCs and STs and other backward

g
classes.

or
3. He lays down the reports of CAG, UPSC before the parliament.

s.
4. He can seek information relating to the administration of the affairs of the union from the PM.

al
Which of the following are the executive powers of the President?

a) 1,2 and 4
ri
b) 2 and 4
c) 1,2 and 3
te

d) 1 and 3
Correct Answer: B
ma

Your Answer: B
Explanation
sc

Explanation: Statements 1 and 3 comes under the legislative powers of the President
up

QUESTION 9.
Consider the following statements regarding Rajya sabha
ee

1. The constitution has fixed the term of office of members of Rajya Sabha as 6 years
fr

2. According to constitution one must be a member of Scheduled caste or Scheduled tribe , if he wants to
contest a seat reserved for them.

Which of the above statements are correct?

a) 1 only
b) 2 only
c) Both 1 and 2
d) Neither 1 nor 2
Correct Answer: D

Civilsdaily
Page 0
Email: hello@civilsdaily.com
Exam Title : TS01 polity test L1
Email : sandeepbiswas221@gmail.com
Contact : 8250416813

Your Answer: A
Explanation

-The constitution has not fixed the term of office of members of Rajya sabha and left it to the Parliament.
Accordingly parliament in Representation of people Act 1951 provided that the the term of office of
members of Rajya Sabha as 6 years.

www.freeupscmaterials.org
-The second statement is also a provision of in Representation of People Act 1951 and not according to the
constitution.

g
or
Ref: Laxmikanth page no 22.4,22.5

s.
QUESTION 10.
Which of the following statements regarding Parliament of India are correct?

al
a) Institution of Speaker and Deputy Speaker originated under Government of Inidia Act 1935.
b) Whenever Speaker is appointed as a member of a parliamentary committee, he automatically becomes
ri
its chairman
c) Speaker of Lok sabha has no security of tenure
te

d) Vice-President cannot vote in the resolution of his removal


Correct Answer: D
ma

Your Answer: D
Explanation
sc

1st statement is wrong because institution of Speaker and Deputy speaker originated under Government of
India Act 1919.
up

2nd statement is wrong because it is not the Speaker but the Deputy speaker whenever appointed as a
ee

member of a parliamentary committee , automatically becomes its chairman


fr

3rd statement is wrong because Speaker of the Lok Sabha is provided with security of tenure. He can be
removed only by a resolution passed by Lok Sabha by an absolute majority.

Ref:Laxmikanth page no:22.8,22.9

QUESTION 11.
Which of the following words are mentioned in the Preamble of the Indian Constitution?

1. Expression

2. Belief

Civilsdaily
Page 0
Email: hello@civilsdaily.com
Exam Title : TS01 polity test L1
Email : sandeepbiswas221@gmail.com
Contact : 8250416813

3. Unity and integrity

4.Opportunity

Select the correct code

a) 3 and 4

www.freeupscmaterials.org
b) 3 only
c) 1, 3 and 4

g
d) 1, 2, 3 and 4

or
Correct Answer: D
Your Answer: D

s.
Explanation

-The Preamble in its present form reads: al


We, THE PEOPLE OF INDIA, having solemnly resolved to constitute India into a SOVEREIGN SOCIALIST
ri
SECULAR DEMOCRATIC REPUBLIC and to secure to all its citizens: JUSTICE, Social, Economic and
Political; LIBERTY of thought, expression, belief, faith and worship; EQUALITY of status and of opportunity;
te

and to promote among them all; FRATERNITY assuring the dignity of the individual and the unity and
integrity of the Nation; IN OUR CONSTITUENT ASSEMBLY this twenty-sixth day of November, 1949, do
ma

HEREBY ADOPT, ENACT AND GIVE TO OURSELVES THIS CONSTITUTION.


sc

QUESTION 12.
Consider the following statements about the Fundamental Rights:
up

1. The Fundamental Rights are meant for promoting the ideal of social and economic democracy.
ee

2. They protect the liberties and freedoms of the people against the invasion by the State.

3. They aim at establishing a government of men.


fr

Which of the above statements are correct?

a) 2 only
b) 2 and 3 only
c) 1 and 2 only
d) 1, 2 and 3
Correct Answer: A
Your Answer: A
Explanation

Civilsdaily
Page 0
Email: hello@civilsdaily.com
Exam Title : TS01 polity test L1
Email : sandeepbiswas221@gmail.com
Contact : 8250416813

-The Fundamental Rights are meant for promoting the ideal of political democracy. DPSPs aim at
promoting the social and economic democracy.

-They prevent the establishment of an authoritarian and despotic rule in the country, and protect the
liberties and freedoms of the people against the invasion by the State.

-They aim at establishing a government of laws and not of men.

www.freeupscmaterials.org
[Source: Chapter 7: Indian Polity: M Laxmikanth]

g
or
QUESTION 13.
Consider the following statements

s.
1. The legislative assembly can only discuss on the demands for grants and is not authorised to vote.

al
2. The legislative assembly can participate in the elections of Vice President of India.

Which of the statements given above is/are correct?


ri

a) Both 1 and 2
te

b) Only 1
c) Only 2
ma

d) Neither 1 nor 2

[Source: Chapter 29: Indian Polity: M Laxmikanth]


sc

Correct Answer: D
Your Answer: D
up

Explanation
ee

The legislative assembly can both discuss and vote on the demands for grants. It is the legislative council
which can only discuss the demand for grants and is not authorised to vote.
fr

-Only the members of both houses of the Parliament participate in the elections to the Vice president of
India.

-The legislative assembly can participate in the elections of President of India.

QUESTION 14.
The tenure of Rajya Sabha which is 6 years has been fixed by

a) Article 82
b) The British conventions
c) The President of India
d) Parliament through legislation

Civilsdaily
Page 0
Email: hello@civilsdaily.com
Exam Title : TS01 polity test L1
Email : sandeepbiswas221@gmail.com
Contact : 8250416813

Correct Answer: D
Your Answer: D
Explanation

-The Constitution has not fixed the term of office of members of the Rajya Sabha and left it to the
Parliament. Accordingly, the Parliament in the Representation of the People Act (1951) provided that the
term of office of a member of the Rajya Sabha shall be six years.

www.freeupscmaterials.org
[Source: Chapter 22: Indian Polity: M Laxmikanth]

g
or
QUESTION 15.
The union executive does not consist of:

s.
a) President
b) Council of Ministers
c) Attorney General of India
al
d) Leader of opposition
ri
Correct Answer: D
Your Answer: D
te

Explanation
ma

-The Union Executive consists of President, Vice President, Prime Minister, Council of Ministers and the
Attorney General of India.
sc

-The state executive consists of the governor, the chief minister, the council of ministers and the advocate
up

general of the state. Thus, there is no office of vice-governor (in the state) like that of Vice-President at the
Centre.
ee

-Leader of opposition is not part of Union executive


fr

QUESTION 16.
In which of the following conditions the state legislature is empowered to make laws?

1. Prescribing residence as a condition for certain employments or appointments in a state or union territory
or local authority.

2. Restricting or abrogating the application of Fundamental Rights to members of armed forces, Police
forces etc.

Select the correct code:

a) Both 1 and 2
b) Only 1
c) Only 2
d) Neither 1 nor 2

Civilsdaily
Page 0
Email: hello@civilsdaily.com
Exam Title : TS01 polity test L1
Email : sandeepbiswas221@gmail.com
Contact : 8250416813

Correct Answer: D
Your Answer: Unanswered
Explanation

-In both the above conditions, only the Parliament is empowered to make laws and not the state legislature.

-Article 35 lays down that the power to make laws, to give effect to certain specified fundamental rights

www.freeupscmaterials.org
shall vest only in the Parliament and not in the state legislatures. In this direction, Article 35 contains the
following provisions:

g
or
1. The Parliament shall have (and the legislature of a state shall not have) power to make laws with respect
to the following matters:

s.
(a) Prescribing residence as a condition for certain employments or appointments in a state or union
territory or local authority or other authority (Article 16). al
(b) Empowering courts other than the Supreme Court and the high courts to issue directions, orders and
ri
writs of all kinds for the enforcement of fundamental rights (Article 32).
te

(c) Restricting or abrogating the application of Fundamental Rights to members of armed forces, police
forces, etc. (Article 33).
ma

(d) Indemnifying any government servant or any other person for any act done during the operation of
martial law in any area (Article 34).
sc

[Source: Chapter 7: Indian Polity: M Laxmikanth]


up

QUESTION 17.
Which of these features have been mentioned in the constitution as a part of its basic structure?
ee

1. Federalism
fr

2. Secularism

3. Judicial Review

4. Liberty

Select the correct code

a) 1 and 2 only
b) 1, 3 and 4 only
c) 2 and 4 only
d) None of the above

Civilsdaily
Page 0
Email: hello@civilsdaily.com
Exam Title : TS01 polity test L1
Email : sandeepbiswas221@gmail.com
Contact : 8250416813

Correct Answer: D
Your Answer: B
Explanation

Even though the basic structure doctrine was given bythe SC, it is yet to define or clarify as to what
constitutes the basic structure of the Constitution.

www.freeupscmaterials.org
It is not mentioned anywhere in the constitution, and our understanding of the basic structure comes from
the various judgements of the court.

g
or
So, all statements are wrong.

s.
The following are part of basic features of the Constitution

Supremacy of the Constitution; Sovereign, democratic andrepublican nature of the Indian polity;
Secular character of the Constitution
al
ri
Separation of powers between the legislature, the executive and thejudiciary; Federal character of
the Constitution; Unity and integrityof the nation; Welfare state
te

Judicial review; Freedom and dignity of the individual;Parliamentary system; Rule of law; Harmony
ma

and balance between Fundamental Rights and Directive Principles; Principle of equality

Free and fair elections; Independence of Judiciary; Limited powerof Parliament to amend the
sc

Constitution; Effective access tojustice; Principle of reasonableness; Powers of the Supreme Court under
Articles 32, 136, 141 and 142.
up

[Source: Chapter 11: Indian Polity: M Laxmikanth]


ee

QUESTION 18.
Consider the following statements about untouchability:
fr

1. A person convicted of the offence of untouchability is disqualified for election to the Parliament or
state legislature.

2. It has been defined under article 17.

Which of the statements given above is/are correct?

a) Both 1 and 2
b) Only 1
c) Only 2
d) Neither 1 nor 2

Civilsdaily
Page 0
Email: hello@civilsdaily.com
Exam Title : TS01 polity test L1
Email : sandeepbiswas221@gmail.com
Contact : 8250416813

Correct Answer: B
Your Answer: A
Explanation

-A person convicted of the offence of untouchability is disqualified for election to the Parliament or state
legislature.

www.freeupscmaterials.org
-It has not been defined anywhere in constitution but by an act of parliament.

g
[Source: Chapter 7: Indian Polity: M Laxmikanth]

or
QUESTION 19.

s.
Consider the following about right to freedom of religion:

1. Article 27 lays down that no person shall be compelled to pay any fee for the promotion or maintenance
of any particular religion or religious denomination.
al
ri
2. The right to freedom of religion is available only to citizens of India.
te

Which of the statements given above is/are correct?

a) 1 only
ma

b) 2 only
c) Both 1 and 2
sc

d) Neither 1 nor 2
Correct Answer: D
up

Your Answer: D
Explanation
ee

-Article 27 lays down that no person shall be compelled to pay any taxes for the promotion or maintenance
fr

of any particular religion or religious denomination

-This provision prohibits only levy of a tax and not a fee. This is because the purpose of a fee is to control
secular administration of religious institutions and not to promote or maintain religion. Thus, a fee can be
levied on pilgrims to provide them some special service or safety measures. Similarly, a fee can be levied
on religious endowments for meeting the regulation expenditure.

- Articles 15, 16, 19, 29 and 30 are available only to citizens. While other articles are available to foreigners
as well.

QUESTION 20.
Which of the following are the major features of parliamentary government in India?

Civilsdaily
Page 0
Email: hello@civilsdaily.com
Exam Title : TS01 polity test L1
Email : sandeepbiswas221@gmail.com
Contact : 8250416813

1. Collective responsibility of the executive to the legislature

2. Leadership of the prime minister

3. Complete separation of the legislature and the executive organs

Select the correct code

www.freeupscmaterials.org
a) 1 and 2 only

g
b) 2 and 3 only

or
c) 1 and 3 only
d) 1, 2 and 3
Correct Answer: A

s.
Your Answer: A
Explanation al
The parliamentary system is based on the principle of cooperation and co-ordination between the
ri
legislative and executive organs while the presidential system is based on the doctrine of separation of
powers between the two organs. So, 3 is incorrect.
te

Major features of Indian Parliamentary govt are:


ma

(a) Presence of nominal and real executives;


sc

(b) Majority party rule,


up

(c) Collective responsibility of the executive to the legislature,

(d) Membership of the ministers in the legislature,


ee

(e) Leadership of the prime minister or the chief minister,


fr

(f) Dissolution of the lower House (Lok Sabha or Assembly).

[Source: Chapter 12: Indian Polity: M Laxmikanth]

QUESTION 21.
In regard to which of the following laws can be made by State legislatures to get them enforced?

1. Fundamental Rights

2. Directive Principles of State Policy

Civilsdaily
Page 0
Email: hello@civilsdaily.com
Exam Title : TS01 polity test L1
Email : sandeepbiswas221@gmail.com
Contact : 8250416813

3. Fundamental Duties

Select the correct code

a) 1 and 2
b) 2 and 3
c) 1 and 3

www.freeupscmaterials.org
d) 1, 2 and 3
Correct Answer: B

g
Your Answer: C

or
Explanation

s.
-Most of the FRs are directly enforceable (self-executed) while a few of them can be enforced on the basis
of a law made for giving effect to them.
al
Such a law can be made only by the Parliament and not by state legislatures so that uniformity throughout
the country is maintained (Article 35). So, 1 is incorrect.
ri

-In regard to 2 and 3 laws can be made by state legislature.


te

[Source: Chapter 7: Indian Polity: M Laxmikanth]


ma

QUESTION 22.
Consider the following statements about DPSPs
sc

1. They have legal sanctions.


up

2. They are non-justiciable in nature.


ee

3. The courts can declare a law violative of any DPSP as unconstitutional and invalid.
fr

Which of the statements given above are not correct?

a) 2 and 3
b) 1 and 3
c) 2 only
d) 1 and 2
Correct Answer: B
Your Answer: B
Explanation

-The statements 1 and 3 are correct about fundamental rights and not DPSPs.

Civilsdaily
Page 0
Email: hello@civilsdaily.com
Exam Title : TS01 polity test L1
Email : sandeepbiswas221@gmail.com
Contact : 8250416813

-The DPSPs have moral and political sanctions and not legal sanctions.

-Also the courts are barred from declaring a law violative of any DPSP as unconstitutional and invalid.
However, they can uphold validity of a law on the ground that it was enacted to give effect to a directive.

[Source: Chapter 8: Indian Polity: M Laxmikanth]

www.freeupscmaterials.org
QUESTION 23.
The Menaka Gandhi case (1978) is famously known for

g
a) Wider interpretation of the Article 21

or
b) Judicious rebalancing of fundamental rights and directiveprinciples
c) Evolution of the rarest of rare doctrine

s.
d) Making the decision of presiding officer in relation to anti defection subject to judicial review.
Correct Answer: A
Your Answer: A
al
Explanation
ri

-In the Menaka case (1978), the Supreme Court overruled its judgement in the Gopalan case by taking a
te

wider interpretation of the Article 21.


ma

QUESTION 24.
The emoluments, allowances, privileges and so on of President can be altered by:
sc

a) A Constitutional Amendment Bill passed by simple majority of Parliament.


b) A Constitutional Amendment Bill passed by special majority of Parliament.
up

c) A Constitutional Amendment Bill passed by special majority of the Parliament and ratified by half of the
state legislatures.
ee

d) By a normal legislative process that does not require Constitutional Amendment.


Correct Answer: A
fr

Your Answer: Unanswered


Explanation

-The emoluments, allowances, privileges, etc. of President are defined in the Second Schedule which can
be amended by a constitutional amendment through Simple majority only.

[Source: Chapter 10: Indian Polity: M Laxmikanth]

QUESTION 25.
In the order of precedence, the speaker has a rank equal to

a) Deputy Prime-Minister
b) Prime Minister
c) Cabinet Ministers
d) Chief Justice of India.

Civilsdaily
Page 0
Email: hello@civilsdaily.com
Exam Title : TS01 polity test L1
Email : sandeepbiswas221@gmail.com
Contact : 8250416813

Correct Answer: D
Your Answer: D
Explanation

The speaker is given a very high position in the order of precedence. He is placed at sixth rank, along with
the Chief Justice of India. This means, he has a higher rank than all cabinet ministers, except the Prime
Minister or Deputy Prime Minister.

www.freeupscmaterials.org
[Source: Chapter 22: Indian Polity: M Laxmikanth]

g
or
QUESTION 26.
Which of the following takes the oath to uphold the constitution and the laws?

s.
1. President

2. CAG
al
ri
3. Supreme court Judge
te

Select the correct answer using the codes given below.

a) 1 and 2 only
ma

b) 2 only
c) 3 only
sc

d) 2 and 3 only
Correct Answer: D
up

Your Answer: D
Explanation
ee

-President takes an oath to preserve, protect and defend the constitution.


fr

- CAG and Supreme court Judge take similar oaths to uphold the constitution and the laws.

-Before entering upon his office, the President has to make and subscribe to an oath or affirmation. In his
oath, the President swears:

1. to faithfully execute the office;

2. to preserve, protect and defend the Constitution and the law; and

3. to devote himself to the service and well-being of the people of India

Civilsdaily
Page 0
Email: hello@civilsdaily.com
Exam Title : TS01 polity test L1
Email : sandeepbiswas221@gmail.com
Contact : 8250416813

-A person appointed as a judge of the Supreme Court, before entering upon his Office, has to make and
subscribe an oath or affirmation before the President, or some person appointed by him for this purpose.

1. to bear true faith and allegiance to the Constitution of India;

www.freeupscmaterials.org
2. to uphold the sovereignty and integrity of India;

g
3. to duly and faithfully and to the best of his ability, knowledge and judgement perform the duties of the

or
Office without fear or favour, affection or ill-will; and

4. to uphold the Constitution and the laws.

s.
[Source: Chapter 30: Indian Polity: M Laxmikanth]

QUESTION 27.
al
Which of the following provisions of Indian constitution came into immediate effect from 26 November,
ri
1949 that is date of enactment of Preamble?
te

1. Emergency provisions
ma

2. Citizenships

3. Elections
sc

4. Federal provisions
up

Select the correct code


ee

a) 1 and 3
b) 2 and 3
fr

c) 1 and 4
d) 2 and 4
Correct Answer: B
Your Answer: B
Explanation

-Some provisions of the Constitution pertaining to citizenship, elections, provisional parliament, temporary
and transitional provisions, and short title contained in Articles 5, 6, 7, 8, 9, 60, 324, 366, 367, 379, 380,
388, 391, 392 and 393 came into force on November 26, 1949 itself.

Civilsdaily
Page 0
Email: hello@civilsdaily.com
Exam Title : TS01 polity test L1
Email : sandeepbiswas221@gmail.com
Contact : 8250416813

-The remaining provisions (the major part) of the Constitution came into force on January 26, 1950. This
day is referred to in the Constitution as the date of its commencement, and celebrated as the Republic
Day.

[Source: Chapter 2: Indian Polity: M Laxmikanth]

QUESTION 28.

www.freeupscmaterials.org
With Regard to Constitutional Amendment Bill which of the following statement is correct

g
a) The President can reject the bill but cannot return the bill.

or
b) The President cannot reject the bill but can return the bill.
c) The President can neither reject the bill nor return the bill.
d) The President can either reject the bill or return the bill.

s.
Correct Answer: C
Your Answer: C
Explanation
al
ri
-After 24th amendment, 1971 the President must give his assent to the bill. He can neither withhold his
assent to the bill nor return the bill for reconsideration of the Parliament.
te

[Source: Chapter 10: Indian Polity: M Laxmikanth]


ma

QUESTION 29.
During which of the following conditions Fundamental Rights guaranteed by the Indian constitution can be
sc

suspended?
up

a) During a proclamation of National Emergency


b) Am amendment of the constitution
ee

c) An act passed by the parliament


d) By the decisions of Supreme Court and High Courts
fr

Correct Answer: A
Your Answer: A
Explanation

-Articles 358 and 359 describe the effect of a National Emergency on the Fundamental Rights. Article 358
deals with the suspension of the Fundamental Rights guaranteed by Article 19, while Article 359 deals with
the suspension of other Fundamental Rights (except those guaranteed by Articles 20 and 21).

-(a) Suspension of Fundamental Rights under Article 19 According to Article 358, when a proclamation of
national emergency is made, the six Fundamental Rights under Article 19 are automatically suspended. No
separate order for their suspension is required.

Civilsdaily
Page 0
Email: hello@civilsdaily.com
Exam Title : TS01 polity test L1
Email : sandeepbiswas221@gmail.com
Contact : 8250416813

-(b) Suspension of other Fundamental Rights Article 359 authorises the presi-dent to suspend the right to
move any court for the enforcement of Fundamental Rights during a National Emergency. This means that
under Article 359, the Fundamental Rights as such are not suspended, but only their enforcement.

[Source: Chapter 16: Indian Polity: M Laxmikanth]

QUESTION 30.

www.freeupscmaterials.org
Under article 15 of the constitution, for which of the following can special provisions be made by the state
under this article?

g
or
1. Economically backward classes

2. Socially backward classes

s.
3. Women

4. Children
al
ri
Select the correct code
te

a) 1 and 3
b) 2, 3 and 4
ma

c) 1, 3 and 4
d) 1, 2, 3 and 4
sc

Correct Answer: B
Your Answer: B
up

Explanation

-Article 15 provides that


ee

(a) The state is permitted to make any special provision for women and children. For example, reservation
fr

of seats for women in local bodies or provision of free education for children.

(b) The state is permitted to make any special provision for the advancement of any socially and
educationally backward classes of citizens or for the scheduled castes and scheduled tribes. For example,
reservation of seats or fee concessions in public educational institutions.

(c) The state is empowered to make any special provision for the advancement of any socially and
educationally backward classes of citizens or for the scheduled castes or the scheduled tribes regarding
their admission to educational institutions including private educational institutions, whether aided or
unaided by the state, except the minority educational institutions.

Civilsdaily
Page 0
Email: hello@civilsdaily.com
Exam Title : TS01 polity test L1
Email : sandeepbiswas221@gmail.com
Contact : 8250416813

[Source: Chapter 7: Indian Polity: M Laxmikanth]

QUESTION 31.
The term equal pay for equal work is

a) Constitutional right
b) Fundamental right

www.freeupscmaterials.org
c) Legal right
d) Directive Principle of State Policy

g
Correct Answer: D

or
Your Answer: A
Explanation

s.
-Article 39 directs the state to secure

(a) the right to adequate means of livelihood for all citizens;


al
ri
(b) the equitable distribution of material resources of the community for the common good;
te

(c) prevention of concentration of wealth and means of production;

(d) equal pay for equal work for men and women;
ma

(e) preservation of the health and strength of workers and children against forcible abuse; and
sc

(f) opportunities for healthy development of children (Article 39).


up

[Source: Chapter 8: Indian Polity: M Laxmikanth]


ee

QUESTION 32.
Which of the following given below is not a fundamental duty according to the Indian constitution?
fr

a) To respect the national anthem


b) To protect monuments and places of national interest
c) To safeguard public property
d) To protect and improve the natural environment
Correct Answer: B
Your Answer: B
Explanation

-(b) is a DPSP.

Civilsdaily
Page 0
Email: hello@civilsdaily.com
Exam Title : TS01 polity test L1
Email : sandeepbiswas221@gmail.com
Contact : 8250416813

-Article 49 directs the state to protect monuments, places and objects of artistic or historic interest which
are declared to be of national importance.

-Fundamental duties are

According to Article 51 A, it shall be the duty of every citizen of India:

www.freeupscmaterials.org
(a) to abide by the Constitution and respect its ideals and institutions, the National Flag and the National
Anthem;

g
or
(b) to cherish and follow the noble ideals that inspired the national struggle for freedom;

(c) to uphold and protect the sovereignty, unity and integrity of India;

s.
(d) to defend the country and render national service when called upon to do so;
al
(e) to promote harmony and the spirit of common brotherhood amongst all the people of India transcending
ri
religious, linguistic and regional or sectional diversities and to renounce practices derogatory to the dignity
of women;
te

(f) to value and preserve the rich heritage of the countrys composite culture;
ma

(g) to protect and improve the natural environment including forests, lakes, rivers and wildlife and to have
compassion for living creatures;
sc

(h) to develop scientific temper, humanism and the spirit of inquiry and reform;
up

(i) to safeguard public property and to abjure violence;


ee

(j) to strive towards excellence in all spheres of individual and collective activity so that the nation
constantly rises to higher levels of endeavour and achievement; and
fr

(k) to provide opportunities for education to his child or ward between the age of six and fourteen years.
This duty was added by the 86th Constitutional Amendment Act, 2002.

QUESTION 33.
In which of the following the office of the Leader of the opposition is mentioned?

a) Constitution of India
b) Rules of the house
c) A separate Parliamentary Statute
d) None of the above
Correct Answer: C

Civilsdaily
Page 0
Email: hello@civilsdaily.com
Exam Title : TS01 polity test L1
Email : sandeepbiswas221@gmail.com
Contact : 8250416813

Your Answer: Unanswered


Explanation

- The leader of the largest Opposition party having not less than one-tenth seats of the total strength of the
House is recognised as the leader of the Opposition in that House

-Though the offices of the leader of the House and the leader of the Opposition are not mentioned in the

www.freeupscmaterials.org
Constitution of India, they are mentioned in the Rules of the House and Parliamentary Statute respectively.

g
[Source: Chapter 22: Indian Polity: M Laxmikanth]

or
QUESTION 34.
Consider the following statements with regard to Calling Attention motion

s.
1. It is introduced in the Parliament to draw attention of the house to a matter of urgent public importance.
al
2. It is an Indian innovation in the parliamentary procedure and is not mentioned in the Rules of Procedure.
ri
Which of the statements given above is/are correct?
te

a) 1 only
b) 2 only
ma

c) Both 1 and 2
d) Neither 1 nor 2
sc

Correct Answer: D
Your Answer: Unanswered
up

Explanation
ee

-Calling Attention Motion is introduced in the Parliament by a member to call the attention of a minister to a
matter of urgent public importance, and to seek an authoritative statement from him on that matter.
fr

-Like the zero hour, it is also an Indian innovation in the parliamentary procedure and has been in existence
since 1954. However, unlike the zero hour, it is mentioned in the Rules of Procedure.

[Source: Chapter 22: Indian Polity: M Laxmikanth]

QUESTION 35.
Which of the following conditions given below may lead to the resignation of the government?

1. Defeat of a money bill.

2. Passing of Censure Motion.

Civilsdaily
Page 0
Email: hello@civilsdaily.com
Exam Title : TS01 polity test L1
Email : sandeepbiswas221@gmail.com
Contact : 8250416813

3. Passing of Cut Motion.

Select the correct code

a) 1 and 2
b) 1 and 3
c) 2 and 3

www.freeupscmaterials.org
d) 1, 2 and 3
Correct Answer: B

g
Your Answer: D

or
Explanation

s.
-Options 1 and 3 lead to defeat of the government.

-Censure Motion al
1. It should state the reasons for its adoption in the Lok Sabha.
ri

2. It can be moved against an individual minister or a group of ministers or the entire council of ministers.
te

3. It is moved for censuring the council of ministers for specific policies and actions.
ma

4. If it is passed in the Lok Sabha, the council of ministers need not resign from the office.
sc

-Defeat of a money bill means the government is in minority in the house. Passing of a cut motion means
that the policy of the government has failed to pass and it may lead to its resignation.
up

[Source: Chapter 22: Indian Polity: M Laxmikanth]


ee

QUESTION 36.
Consider the following statements with regard to Ordinance making power of President
fr

1. The President can promulgate an ordinance to amend tax laws.

2. The President can promulgate an ordinance to amend the constitution.

Which of the statements given above is/are correct?

a) 1 only
b) 2 only
c) Both 1 and 2
d) Neither 1 nor 2

Civilsdaily
Page 0
Email: hello@civilsdaily.com
Exam Title : TS01 polity test L1
Email : sandeepbiswas221@gmail.com
Contact : 8250416813

Correct Answer: A
Your Answer: D
Explanation

-Article 123 of the Constitution empowers the President to promulgate ordinances during the recess of
Parliament. These ordinances have the same force and effect as an act of Parliament.

www.freeupscmaterials.org
-His ordinance-making power is coextensive as regards all matters except duration, with the law-making
powers of the Parliament.

g
or
-An ordinance like any other legislation, can be retrospective, that is, it may come into force from a back
date. It may modify or repeal any act of Parliament or another ordinance. It can alter or amend a tax law

s.
also. However, it cannot be issued to amend the Constitution.

[Source: Chapter 17: Indian Polity: M Laxmikanth] al


QUESTION 37.
ri
Given below are some of the expenditures which are incurred on various functions. Which of the following
Expenditures is/are the expenditure Charged on the Consolidated fund of India?
te

1. Emoluments and allowances of President & Vice President.


ma

2. Salaries, allowances and pensions of the Judges of the Supreme Court & high court.
sc

3. Salaries, allowances and pension of the CAG.


up

4. Salaries, allowances and pensions of the chairman and members of UPSC.

Select the correct code


ee

a) 1, 2 and 3
fr

b) 2, 3 and 4
c) 1, 3 and 4
d) 1, 2, 3 and 4
Correct Answer: C
Your Answer: Unanswered
Explanation

-The budget consists of two types of expenditure- the expenditure charged upon the Consolidated Fund of
India and the expenditure made from the Consolidated Fund of India. The charged expenditure is
non-votable by the Parliament, that is, it can only be discussed by the Parliament, while the other type has

Civilsdaily
Page 0
Email: hello@civilsdaily.com
Exam Title : TS01 polity test L1
Email : sandeepbiswas221@gmail.com
Contact : 8250416813

to be voted by the Parliament.

- The salaries and other expenses of the judges and maintenance of the state high courts are charged from
Consolidated fund of the state. However please note that the retired Judges are entitled to a pension which
is drawn from consolidated fund of India.

QUESTION 38.

www.freeupscmaterials.org
The Constitution of India lays which of the following qualification/s for a person to be appointed as the
Governor of a state?

g
or
1. He should be a citizen of India.

2. Must have completed the age of 35 years.

s.
3. He shall not belong to the state where he is appointed.

a) 1 only.
al
ri
b) 1 and 2
c) 1, 2 and 3
d) 2 and 3
te

Correct Answer: B
Your Answer: B
ma

Explanation
sc

-The Constitution lays down only two qualifications for the appointment of a person as a governor. These
are:
up

1. He should be a citizen of India.


ee

2. He should have completed the age of 35 years.


fr

-Additionally, two conventions have also developed in this regard over the years. First, he should be an
outsider, that is, he should not belong to the state where he is appointed, so that he is free from the local
politics. Second, while appointing the governor, the president is required to consult the chief minister of the
state concerned, so that the smooth functioning of the constitutional machinery in the state is ensured.

[Source: Chapter 26: Indian Polity: M Laxmikanth]

QUESTION 39.
Consider the following statements with regard to Panchayati Raj Institutions

Civilsdaily
Page 0
Email: hello@civilsdaily.com
Exam Title : TS01 polity test L1
Email : sandeepbiswas221@gmail.com
Contact : 8250416813

1. All members of Panchayat at the village, intermediate and district levels are elected directly by the
people.

2. Governor can remove the State Election Commissioner on the recommendation of the State Legislature.

3. The 73rd Amendment act of 1992 is applicable to all states except J&K.

www.freeupscmaterials.org
Which of the statements given above is/are correct?

g
a) 1 only

or
b) 1 and 3
c) 2 and 3
d) 1, 2 and 3

s.
Correct Answer: A
Your Answer: Unanswered
Explanation
al
ri
All the members of panchayats at the village, intermediate and district levels
te

-Election of Members and Chairpersons shall be elected directly by the people. Further, the chairperson of
panchayats at the intermediate and district levels shall be elected indirectlyby and from amongst the
ma

elected members thereof. However, the chairperson of a panchayat at the village level shall be elected in
such manner as the state legislature determines.
sc

-The super-intendence, direction and control of the preparation of electoral rolls and the conduct of all
elections to the panchayats shall be vested in the state election commission. It consists of a state election
up

commissioner to be appointed by the governor. His conditions of service and tenure of office shall also be
determined by the governor. He shall not be removed from the office except in the manner and on the
ee

grounds prescribed for the removal of a judge of the state high court.
fr

-Exempted States and Areas The act does not apply to the states of Jammu and Kashmir, Nagaland,
Meghalaya and Mizoram and certain other areas. These areas include, (a) the scheduled areas and the
tribal areas in the states; (b) the hill area of Manipur for which a district council exists; and (c) Darjeeling
district of West Bengal for which Darjeeling Gorkha Hill Council exists.

[Source: Chapter 34: Indian Polity: M Laxmikanth]

QUESTION 40.
The constitution of India has borrowed which of the following features from GOI Act 1935?

1. Office of the Governor.

Civilsdaily
Page 0
Email: hello@civilsdaily.com
Exam Title : TS01 polity test L1
Email : sandeepbiswas221@gmail.com
Contact : 8250416813

2. Emergency Provisions.

3. Public Service Commissions

4. Bicameralism.

5. Federation with a strong center.

www.freeupscmaterials.org
a) 1, 2 and 4

g
b) 1, 2 and 3

or
c) 1, 2, 4 and 5
d) 1, 2, 3, 4 and 5
Correct Answer: B

s.
Your Answer: D
Explanation al
-Features borrowed from Government of India Act 1935
ri

Federal Scheme (also from constitution of Canada)


te

Office of Governor
ma

Judiciary
sc

Public Service Commission


up

Emergency Provisions

Administrative Details
ee

QUESTION 41.
fr

Consider the following statements:

1. Supreme court can refuse to exercise its writ jurisdiction.

2. The writ jurisdiction of SC is wider than that of a High Court.

Which of the statements given above is/are correct?

a) 1 only
b) 2 only
c) Both 1 and 2
d) Neither 1 nor 2

Civilsdaily
Page 0
Email: hello@civilsdaily.com
Exam Title : TS01 polity test L1
Email : sandeepbiswas221@gmail.com
Contact : 8250416813

Correct Answer: D
Your Answer: D
Explanation

-The writ jurisdiction of the Supreme Court differs from that of a high court in three respects:

1. The Supreme Court can issue writs only for the enforcement of fundamental rights whereas a high court

www.freeupscmaterials.org
can issue writs not only for the enforcement of Fundamental Rights but also for any other purpose. The
expression for any other purpose refers to the enforcement of an ordinary legal right. Thus, the writ

g
jurisdiction of the Supreme Court, in this respect, is narrower than that of high court.

or
2. A remedy under Article 32 is in itself a Fundamental Right and hence, the Supreme Court cannot refuse

s.
to exercise its writ jurisdiction. On the other hand, a remedy under Article 226 is discretionary and hence, a
high court may refuse to exercise its writ jurisdiction. Article 32 does not merely confer power on the
al
Supreme Court as Article 226 does on a high court to issue writs for the enforcement of fundamental rights
or other rights as part of its general jurisdiction. The Supreme Court is thus constituted as a defender and
ri
guarantor of the fundamental rights.

[Source: Chapter 7: Indian Polity: M Laxmikanth]


te

QUESTION 42.
ma

In which of the following ways the Indian parliamentary system of government differs from that of the British
parliamentary system?
sc

1. The Indian system is based on the doctrine of the sovereignty of Parliament, while the Parliament is not
supreme in Britain.
up

2. In Britain, the prime minister should be a member of the Lower House of the Parliament, whereas this is
ee

not compulsory in India.


fr

Select the correct code

a) 1 only
b) 2 only
c) Both 1 and 2
d) Neither 1 nor 2
Correct Answer: B
Your Answer: B
Explanation

Civilsdaily
Page 0
Email: hello@civilsdaily.com
Exam Title : TS01 polity test L1
Email : sandeepbiswas221@gmail.com
Contact : 8250416813

-Indian Parliament enjoys limited and restricted powers dueto a written constitution. It is based on
constitutional supremacy. While in Britain, parliament is sovereign.

-In India, the prime minister may be a member of any of thetwo Houses of Parliament. Even a non-member
can become the PMprovided he becomes the member within six months of entering office.

-Moreover, India has a republican system in place of British monarchicalsystem. In other words, the Head

www.freeupscmaterials.org
of the State in India (President)is elected, while the Head of the State in Britain (King or Queen)enjoys a
hereditary position.

g
or
[Source: Chapter 12: Indian Polity: M Laxmikanth]

QUESTION 43.

s.
Consider the following statements

al
1. The Governor can make regulations for the peace and good governance of certain Union Territories like
Andaman and Nicobar, Lakshadweep, Dadar and Nagar Haveli and Daman and Diu.
ri

2. The governor is empowered to direct that an act of Parliament does not apply to a scheduled area in the
state or apply with specified modifications and exceptions.
te

Which of the statements given above is/are correct?


ma

a) 1 only
sc

b) 2 only
c) Both 1 and 2
up

d) None
Correct Answer: B
ee

Your Answer: B
Explanation
fr

The laws of Parliament are not applicable in thefollowing areas:

-The President can make regulations for the peace, progress andgood government of the four Union
Territoriesthe Andaman andNicobar Islands, Lakshadweep, Dadra and Nagar Haveli andDaman and Diu.
A regulation so made has the same force andeffect as an act of Parliament. It may also repeal or amend
any actof Parliament in relation to these union territories.

-The governor is empowered to direct that an act of Parliament doesnot apply to a scheduled area in the
state or apply with specified modifications and exceptions.

Civilsdaily
Page 0
Email: hello@civilsdaily.com
Exam Title : TS01 polity test L1
Email : sandeepbiswas221@gmail.com
Contact : 8250416813

-The Governor of Assam may likewise direct that an act of Parliament does not apply to a tribal area
(autonomous district) inthe state or apply with specified modifications and exceptions. The President enjoys
the same power with respect to tribal areas (autonomous districts) in Meghalaya, Tripura and Mizoram.

QUESTION 44.
For subjects in the concurrent list, the executive power rests with:

www.freeupscmaterials.org
a) Centre government
b) State government

g
c) Supreme Court

or
d) Lok Sabha
Correct Answer: B

s.
Your Answer: A
Explanation
al
-Executive power over laws made on subjects mentioned in the concurrent list doesnt lie with the centre.
ri
-Executive power over laws made on subjects mentioned in the concurrent list lies with the state except
when a constitutional provision or a parliamentary law specifically confers it on the centre.
te

-Restrictions on the use of executive power of the state Executive power of the state should be used in
ma

such a way so

i) as to ensure compliance with the laws made by the Parliament and any existing law which apply in the
sc

state.
up

ii) as not to impede or prejudice the exercise of executive power of the Centre in the state.
ee

QUESTION 45.
According to the Indian Constitution, the advice tendered by the Council of Ministers is binding on the
President. It is a
fr

a) Constitutional provision
b) Statutory provision
c) Convention
d) Executive resolution
Correct Answer: A
Your Answer: A
Explanation

Civilsdaily
Page 0
Email: hello@civilsdaily.com
Exam Title : TS01 polity test L1
Email : sandeepbiswas221@gmail.com
Contact : 8250416813

-The President is the nominal executive (de jure executive) while the Prime Minister is the real executive
(de facto executive). Thus, the President is head of the State, while the PrimeMinister is head of the
government.

-Article 74 provides for a council of ministers headed by the PrimeMinister to aid and advise the President
in the exercise of his functions.The advice so tendered is binding on the President.

www.freeupscmaterials.org
[Source: Chapter 20: Indian Polity: M Laxmikanth]

g
QUESTION 46.

or
The constitution can be amended as per the procedure laid down in Article 368. Consider the following
statements in this regard

s.
1. An amendment of the Constitution can be initiated in either Parliament or State Legislature.

al
2. Concerned bill may be introduced either by a minister or a private member.
ri
3. Prior Presidential recommendation is not necessary to introduce the bill.

4. President may give his assent to the bill or return it back for reconsideration of the House.
te

Which of the statements given above are correct?


ma

a) 1, 2 and 4 only
b) 2 and 3 only
sc

c) 3 and 4 only
d) 2, 3 and 4 only
up

Correct Answer: B
Your Answer: B
ee

Explanation
fr

-An amendment of the Constitution can beinitiated only by the introduction of a bill for the purpose in either
Houseof Parliament and not in the state legislatures.

-The bill can be introduced either by a minister or bya private member and does not require prior
permission of the president.

-The President must give his assent to the bill. He cannot return it back for reconsideration. After 24th
Amendment, 1971 he is bound to give his assent to the Constitutional Amendment Bill.

-Each House must pass the bill separately. In case of a disagreement between the two Houses, there is no
provision for holding a joint sittingof the two Houses for the purpose of deliberation and passage of the bill.

Civilsdaily
Page 0
Email: hello@civilsdaily.com
Exam Title : TS01 polity test L1
Email : sandeepbiswas221@gmail.com
Contact : 8250416813

[Source: Chapter 10: Indian Polity: M Laxmikanth]

QUESTION 47.
In which of the following ways a resolution of disapproval of national emergency is different from a
resolution approving the continuation of a proclamation?

1. Disapproval of emergency requires the approval of both the houses, whereas approval of proclamation

www.freeupscmaterials.org
can be done only by the Lok Sabha.

g
2. Disapproval requires special majority, whereas approval requires a simple majority.

or
Select the correct code

s.
a) 1 only
b) 2 only
c) Both 1 and 2
d) Neither 1 nor 2
al
ri
Correct Answer: D
Your Answer: Unanswered
te

Explanation
ma

-Statement 1- First one (disapproval) is required to bepassed by the Lok Sabha only, while the second one
needs to be passedby the both Houses of Parliament.
sc

-Statement 2- The first one is to be adopted by a simple majority only,while the second one needs to be
adopted by a special majority.
up

[Source: Chapter 16: Indian Polity: M Laxmikanth]


ee

QUESTION 48.
Q.48 Consider the following statements:
fr

1. If Lok Sabha passes a resolution that it is necessary in national interest that Parliament should make
laws on a state subject, Parliament can enact laws on that subject.

2. Such a resolution must be supported by a simple majority

3. The resolution remains in force for a period of indefinite time.

Which of the statements given above are correct?

a) 1 only
b) 2 only
c) 3 only
d) None of the above

Civilsdaily
Page 0
Email: hello@civilsdaily.com
Exam Title : TS01 polity test L1
Email : sandeepbiswas221@gmail.com
Contact : 8250416813

Correct Answer: D
Your Answer: Unanswered
Explanation

-All the statements are incorrect.

-If RajyaSabha and not Lok Sabha passes a resolution that it is necessary in national interest that

www.freeupscmaterials.org
Parliament should make laws on a state subject, Parliament can enact laws on that subject.

g
-Such a resolution must be supported by two-third of the members present and voting.

or
-The resolution remains in force for one year, it can be renewed any number of times but not exceeding

s.
one year at a time.

-The law ceases to have effect on the expiration of six months after the resolution has ceased to be in
force.
al
ri
[Source: Chapter 14: Indian Polity: M Laxmikanth]
te

QUESTION 49.
Consider the following statements about Zonal councils:
ma

1. The zonal councils are constitutional bodies established under Art 263.
sc

2. The Prime minister is the common chairman of all five Zonal Councils.

Which of the statements given above is/are correct?


up

a) 1 only
ee

b) 2 only
c) Both 1 and 2
fr

d) Neither 1 nor 2
Correct Answer: D
Your Answer: Unanswered
Explanation

-Article 263 provides for the creation of Inter-state council and not zonal councils.

-The Zonal Councils are the statutory (and not the constitutional) bodies. They are established by an Act of
the Parliament, that is, States Reorganisation Act of 1956. The act divided the country into five zones
(Northern, Central, Eastern, Western and Southern) and provided a zonal council for each zone.

Civilsdaily
Page 0
Email: hello@civilsdaily.com
Exam Title : TS01 polity test L1
Email : sandeepbiswas221@gmail.com
Contact : 8250416813

-Each zonal council consists of the following members: (a) home minister of Central government. (b) chief
ministers of all the States in the zone. (c) Two other ministers from each state in the zone. (d) Administrator
of each union territory in the zone.

-The home minister of Central government is the common chairman of the five zonal councils.

-Each chief minister acts as a vice-chairman of the council by rotation, holding office for a period of one

www.freeupscmaterials.org
year at a time.

g
-The zonal councils aim at promoting cooperation and coordination between states, union territories and

or
the Centre. They discuss and make recommendations regarding matters like economic and social
planning, linguistic minorities, border disputes, inter-state transport, and so on.

s.
-They are only deliberative and advisory bodies.

[Source: Chapter 15: Indian Polity: M Laxmikanth] al


ri
QUESTION 50.
Consider the following about writ of Certiorari
te

1. It is issued by a higher court to a lower court or tribunal either to transfer a case pending with the latter to
itself or to squash the order of the latter in a case.
ma

2. Unike prohibition, certiorari is both preventive and curative.


sc

3. It cannot be issued against administrative authorities affecting rights of individuals.


up

Which of the above statements are correct?


ee

a) 1 and 2
b) 1 and 3
fr

c) 3 only
d) 1, 2 and 3
Correct Answer: A
Your Answer: A
Explanation

-It is issued by a higher court to a lower court or tribunal either to transfer a case pending with the latter to
itself or to squash the order of the latter in a case. It is issued on the grounds of excess of jurisdiction or
lack of jurisdiction or error of law.

-Unlike prohibition, which is only preventive, certiorari is both preventive as well as curative.

Civilsdaily
Page 0
Email: hello@civilsdaily.com
Exam Title : TS01 polity test L1
Email : sandeepbiswas221@gmail.com
Contact : 8250416813

-In 1991, the Supreme Court ruled that the certiorari can be issued even against administrative authorities
affecting rights of individuals.

-Like prohibition, certiorari is also not available against legislative bodies and private individuals or bodies.

[Source: Chapter 7: Indian Polity: M Laxmikanth]

www.freeupscmaterials.org
QUESTION 51.
Consider the following authorities:

g
or
1. Advocate general of a state

2. State election commissioner

s.
3. Chairman and members of the state public service commission

4. Vice-chancellors of universities in the state


al
ri
Which of the above is/are appointed by Governor?
te

a) 1, 2 and 4 only
b) 2 and 3 only
ma

c) 1 and 4 only
d) 1, 2, 3 and 4
sc

Correct Answer: D
Your Answer: D
up

Explanation

Governor appoints the advocate general of a state and determines his remuneration. The advocate general
ee

holds office during the pleasure of the governor.


fr

-He appoints the state election commissioner and determines his conditions of service and tenure of office.
However, the state election commissioner can be removed only in like manner and on the like grounds as a
judge of a high court.

-He appoints the chairman and members of the state public service commission. However, they can be
removed only by the president and not by a governor.

-He acts as the chancellor of universities in the state. He also appoints the vice-chancellors of universities
in the state.

QUESTION 52.

Civilsdaily
Page 0
Email: hello@civilsdaily.com
Exam Title : TS01 polity test L1
Email : sandeepbiswas221@gmail.com
Contact : 8250416813

Consider the following statements w.r.t powers of governor

1. Executive

2. Legislative

3. Financial

www.freeupscmaterials.org
4. Judicial

g
5. Emergency

or
Which of the above is/are possessed by Governor?

s.
a) 1, 2 and 5 only
b) 2 and 3 only
c) 1, 3 and 5 only
al
ri
d) 1, 2, 3 and 4 only
Correct Answer: D
te

Your Answer: D
Explanation
ma

A governor possesses executive, legislative, financial and judicial powers more or less analogous to the
President of India.
sc

However, he has no diplomatic, military or emergency powers like the president.


up

QUESTION 53.
A Federation can be formed by the following ways:
ee

a) By integration
fr

b) By disintegration
c) Both (a) and (b)
d) None of the above
Correct Answer: C
Your Answer: C
Explanation

A Federation can be formed by two ways: by integration and by disintegration. In the first case, a number of
militarily weak or economically backward states coome together to form a big and a strong union, example,
US. In the second case, a big unitary state is converted into a federation by granting autonomy to the

Civilsdaily
Page 0
Email: hello@civilsdaily.com
Exam Title : TS01 polity test L1
Email : sandeepbiswas221@gmail.com
Contact : 8250416813

provinces, example, Canada.

Source: Laxmikanth

QUESTION 54.
Which among the following statement regarding President's Rule is correct?

www.freeupscmaterials.org
a) The State executive and legislature continues to function but the Centre gets simultaneous powers of
administration and legislation in the state.

g
b) The Parliament can delegate the power to make laws for the state to the President or to any other

or
authority specified by him.
c) The fundamental rights may or may not get suspended.
d) It is revoked by the Lok Sabha by passing a resolution

s.
Correct Answer: B
Your Answer: C
Explanation
al
ri
(a) It's a feature of National Emergency not President's rule
te

(c) Unlike National emergency,President's rule has no effect on the Fundamental rights of citizens.
ma

(d) There is no such provision.It can be revoked by the President only on his own.

Ref:Laxmikanth 16.7
sc

QUESTION 55.
up

Consider the following statements regarding CAG of India:

1.The audit of Reserve bank of India,State bank of India and Food Corporation of India are directly done by
ee

CAG.
fr

2.CAG submits three audit reports such as report on appropriation account ,report on finance account,and
audit report on public undertakings to the Parliament.

3.He audits the accounts of any authority when requested by President or Governor

Select the correct codes:

a) 1 only
b) 1 and 2
c) 3 only
d) 2 and 3

Civilsdaily
Page 0
Email: hello@civilsdaily.com
Exam Title : TS01 polity test L1
Email : sandeepbiswas221@gmail.com
Contact : 8250416813

Correct Answer: C
Your Answer: Unanswered
Explanation

1.The audit of Reserve bank of India,State bank of India and Food Corporation of India are totally subjected
to private audit.

www.freeupscmaterials.org
2.CAG submits three audit reports to the President and not to the Parliament.

g
Ref:Laxmikanth page no 45.2,45.3

or
QUESTION 56.

s.
Which of the following statements regarding Parliament of India are correct?

a) Institution of Speaker and Deputy Speaker originated under Government of India Act 1935.
al
b) Whenever Speaker is appointed as a member of a parliamentary committee, he automatically becomes
its chairman
ri
c) Speaker of Lok sabha has no security of tenure
d) Vice-President cannot vote in the resolution of his removal
te

Correct Answer: D
Your Answer: D
ma

Explanation
sc

1st statement is wrong because institution of Speaker and Deputy speaker originated under Government of
Inidia Act 1919.
up

2nd statement is wrong because Its not the speaker but the Deputy speaker whenever appointed as a
member of a parliamentary committee , automatically becomes its chairman
ee

3rd statement is wrong because Speaker of Lok sabha is provided with security of tenure.He can be
fr

removed only by a resolution passed by Lok sabha by an absolute majority.

Ref:Laxmikanth page no:22.8,22.9

QUESTION 57.
Which of the following statements are true about the Governor of state :

1. If a person is elected as the governor of two or more states, the emoluments and allowances payable to
him are shared by the states in such proportion as determined by the parliament.

2.After giving two months notice, civil proceedings can be instituted against him during his term of office in
respect of his personal acts.

Civilsdaily
Page 0
Email: hello@civilsdaily.com
Exam Title : TS01 polity test L1
Email : sandeepbiswas221@gmail.com
Contact : 8250416813

3.The oath of office to the governor is administered by the President of India.

4.The Constitution does not lay down any grounds upon which a governor may be removed by the
President.

Select the correct codes:

www.freeupscmaterials.org
a) 2 and 4
b) 1, 3 and 4

g
c) 3 and 4

or
d) 1 and 2
Correct Answer: A

s.
Your Answer: Unanswered
Explanation
al
If a person is elected as the governor of two or more states, the emoluments and allowances payable to
him are shared by the states in such proportion as determined by the President.
ri

The oath of office to the governor is administered by the Chief Justice of respective High Court.
te

QUESTION 58.
ma

Which of the following statements is/are correct regarding the Office of CAG of India ?

1.It is constitutional office and appointed by PM.


sc

2. It is the guardian and public purse of centre only


up

Select the correct code:


ee

a) only 1
b) only 2
fr

c) both 1 and 2
d) neither 1 or 2
Correct Answer: D
Your Answer: Unanswered
Explanation

CAG is appointed by President on the advice of PM, and it is guardian and public purse of both the centre
and state.

QUESTION 59.
Consider the following statements about the Presidents Rule

Civilsdaily
Page 0
Email: hello@civilsdaily.com
Exam Title : TS01 polity test L1
Email : sandeepbiswas221@gmail.com
Contact : 8250416813

1. The President can take up the functions of the state government and powers vested in the governor or
any other executive authority in the state.

2. He can declare that the powers of the state legislature are to be exercised by the Parliament.

3. He can take all other necessary steps including the suspension of the constitutional provisions relating to
any body or authority in the state including the State High Court.

www.freeupscmaterials.org
Which of the statements given above are correct?

g
or
a) 1 and 2
b) 1 and 3
c) 2 and 3

s.
d) 1, 2 and 3
Correct Answer: A
Your Answer: Unanswered
al
Explanation
ri

-The President acquires the following extraordinary powers when the Presidents Rule is imposed in a
te

state:
ma

1. He can take up the functions of the state government and powers vested in the governor or any other
executive authority in the state.
sc

2. He can declare that the powers of the state legislature are to be exercised by the Parliament.
up

3. He can take all other necessary steps including the suspension of the constitutional provisions relating to
any body or authority in the state.
ee

-President cannot assume to himself the powers vested in the concerned state high court or suspend the
provisions of the Constitution relating to it. In other words, the constitutional position, status, powers and
fr

functions of the concerned state high court remain same even during the Presidents Rule.

[Source: Chapter 16: Indian Polity: M Laxmikanth]

QUESTION 60.
Liberty of thought, expression, belief, faith and worship is a part of

a) DPSP
b) Fundamental duties
c) FRs
d) Preamble of India

Civilsdaily
Page 0
Email: hello@civilsdaily.com
Exam Title : TS01 polity test L1
Email : sandeepbiswas221@gmail.com
Contact : 8250416813

Correct Answer: D
Your Answer: D
Explanation

-The preamble of India reads as:

We, THE PEOPLE OF INDIA, having solemnly resolved to constitute India into a SOVEREIGN

www.freeupscmaterials.org
SOCIALIST SECULAR DEMOCRATIC REPUBLIC and to secure to all its citizens: JUSTICE, Social,
Economic and Political; LIBERTY of thought, expression, belief, faith and worship; EQUALITY of status and

g
of opportunity; and to promote among them all; FRATERNITY assuring the dignity of the individual and the

or
unity and integrity of the Nation; IN OUR CONSTITUENT ASSEMBLY this twenty-sixth day of November,
1949, do HEREBY ADOPT, ENACT AND GIVE TO OURSELVES THIS CONSTITUTION.

s.
[Source: Chapter 7: Indian Polity: M Laxmikanth]

QUESTION 61.
al
Which of the following are features of Federal government?
ri

1. Dual Government
te

2. Written Constitution
ma

3. Supremacy of the parliament


sc

4. Rigid Constitution
up

Select the correct code

a) 1, 2 and 3
ee

b) 1, 2 and 4
c) 2, 3 and 4
fr

d) 1, 2, 3 and 4

[Source: Chapter 13: Indian Polity: M Laxmikanth]


Correct Answer: B
Your Answer: B
Explanation

-Features of Federal Government

1. Dual Government (that is, national government and regional government)

Civilsdaily
Page 0
Email: hello@civilsdaily.com
Exam Title : TS01 polity test L1
Email : sandeepbiswas221@gmail.com
Contact : 8250416813

2. Written Constitution

3. Division of powers between the national and regional government

4. Supremacy of the constitution and not parliament

5. Rigid Constitution

www.freeupscmaterials.org
6. Independent judiciary

g
7. Bicameral legislature

or
QUESTION 62.

s.
Consider the following statements about creation of Legislative council in states

al
1. In the Parliament, motion for the creation or abolition of Legislative Council requires simple majority.

2. If the motion for abolition or creation of Legislative Council is passed, it is deemed as an amendment of
ri
the constitution under Art 368.
te

Which of the statements given above is/are correct?

a) 1 only
ma

b) 2 only
c) Both 1 and 2
sc

d) Neither 1 nor 2
Correct Answer: A
up

Your Answer: Unanswered


Explanation
ee

-The Constitution provides for the abolition or creation of legislative councils in states. Accordingly, the
fr

Parliament can abolish a legislative council (where it already exists) or create it (where it does not exist), if
the legislative assembly of the concerned state passes a resolution to that effect.

-Such a specific resolution must be passed by the state assembly by a special majority, that is, a majority
of the total membership of the assembly and a majority of not less than two-thirds of the members of the
assembly present and voting.

-This Act of Parliament is not to be deemed as an amendment of the Constitution for the purposes of Article
368 and is passed like an ordinary piece of legislation (ie, by simple majority).

[Source: Chapter 29: Indian Polity: M Laxmikanth]

Civilsdaily
Page 0
Email: hello@civilsdaily.com
Exam Title : TS01 polity test L1
Email : sandeepbiswas221@gmail.com
Contact : 8250416813

QUESTION 63.
Which of the following provisions about national emergency are correct?

1. Once enforced the laws made by Parliament on the state subjects during a National Emergency continue
lifelong.

2. While a proclamation of National Emergency is in operation, the life of the Lok Sabha can be extended

www.freeupscmaterials.org
beyond its normal term with the approval of the President.

g
Select the correct code

or
a) 1 only
b) 2 only

s.
c) Both 1 and 2
d) Neither 1 nor 2
Correct Answer: D
al
Your Answer: Unanswered
ri
Explanation
te

-The laws made by Parliament on the state subjects during a National Emergency become inoperative six
months after the emergency has ceased to operate.
ma

-While a proclamation of National Emergency is in operation, the life of the Lok Sabha may be extended
beyond its normal term (five years) by a law of Parliament for one year at a time (for any length of time). It
sc

is not possible to extend the life of Lok Sabha by the approval of President. Parliament passes a resolution
in this regard.
up

[Source: Chapter 16: Indian Polity: M Laxmikanth]


ee

QUESTION 64.
Which of the following can curtail the jurisdiction and powers of Supreme Court?
fr

a) President
b) Parliament
c) Chief Justice of India
d) None of the above
Correct Answer: D
Your Answer: D
Explanation

Civilsdaily
Page 0
Email: hello@civilsdaily.com
Exam Title : TS01 polity test L1
Email : sandeepbiswas221@gmail.com
Contact : 8250416813

-The Parliament is not authorised to curtail the jurisdiction and powers of the Supreme Court. The
Constitution has guaranteed to the Supreme Court, jurisdiction of various kinds. However, the Parliament
can extend the same.

[Source: Chapter 25: Indian Polity: M Laxmikanth]

QUESTION 65.

www.freeupscmaterials.org
Which of the following fall under original jurisdiction of Supreme Court?

g
1. Inter-state water disputes

or
2. Disputes between Centre and one or more states

s.
3. Ordinary disputes of commercial nature between Centre and States

Select the correct code al


a) 1 and 2
ri
b) 2 only
c) 2 and 3
te

d) 1, 2 and 3
Correct Answer: B
ma

Your Answer: D
Explanation
sc

-Original jurisdiction of Supreme Court extends to any dispute between


up

(a) the Centre and one or more states; or


ee

(b) the Centre and any state or states on one side and one or more states on the other; or
fr

(c) between two or more states.

-This jurisdiction of the Supreme Court does not extend to the following:

(a) A dispute arising out of any pre-Constitution treaty, agreement, covenant, engagement, sanad or other
similar instrument.

(b) A dispute arising out of any treaty, agreement, etc., which specifically provides that the said jurisdiction
does not extend to such a dispute.

(c) Inter-state water disputes.

Civilsdaily
Page 0
Email: hello@civilsdaily.com
Exam Title : TS01 polity test L1
Email : sandeepbiswas221@gmail.com
Contact : 8250416813

(d) Matters referred to the Finance Commission.

(e) Adjustment of certain expenses and pensions between the Centre and the states.

(f) Ordinary dispute of Commercial nature between the Centre and the states.

(g) Recovery of damages by a state against the Centre.

www.freeupscmaterials.org
[Source: Chapter 25: Indian Polity: M Laxmikanth]

g
QUESTION 66.

or
Consider the following statements about appeal by special leave

s.
1. It is a discretionary power and hence cannot be claimed as a matter of right.

2. It can be granted only in final judgements. al


3. It can be related to any matter- constitutional, civil, criminal, income tax etc.
ri

Which of the statements given above is/are correct?


te

a) 1 and 2
b) 2 and 3
ma

c) 1 and 3
d) 1, 2 and 3
sc

Correct Answer: C
Your Answer: Unanswered
up

Explanation
ee

-The Supreme Court is authorised to grant in its discretion special leave to appeal from any judgement in
any matter passed by any court or tribunal in the country (except military tribunal and court martial). This
fr

provision contains the four aspects as under:

(i) It is a discretionary power and hence, cannot be claimed as a matter of right.

(ii) It can be granted in any judgement whether final or interlocutory.

(iii) It may be related to any matterconstitutional, civil, criminal, income-tax, labour, revenue, advocates,
etc.

(iv) It can be granted against any court or tribunal and not necessarily against a high court (of course,
except a military court).

Civilsdaily
Page 0
Email: hello@civilsdaily.com
Exam Title : TS01 polity test L1
Email : sandeepbiswas221@gmail.com
Contact : 8250416813

[Source: Chapter 25: Indian Polity: M Laxmikanth]

QUESTION 67.
Consider the following statements about Parliamentary Committees.

1. They are set up by Prime-Minister according to the exigencies of time and requirement of the situation.

www.freeupscmaterials.org
2. They work under the direction of Speaker/ Chairman of House.

g
3. These committees present their report to the President which cause them to be laid before the

or
Parliament.

Which of the statements given above is/are correct?

s.
a) 1 only
b) 1 and 2 only
c) 2 only
al
ri
d) 2 and 3 only
Correct Answer: C
te

Your Answer: Unanswered


Explanation
ma

-A parliamentary committee means a committee that:


sc

1. Is appointed or elected by the House or nominated by the Speaker / Chairman


up

2. Works under the direction of the Speaker / Chairman

3. Presents its report to the House or to the Speaker / Chairman


ee

4. Has a secretariat provided by the LokSabha / RajyaSabha


fr

-The consultative committees, which also consist of members of Parliament, are not parliamentary
committees as they do not fulfill above four conditions.

-The parliamentary committees are of two kindsStanding Committees and Ad Hoc Committees. The
former are permanent (constituted every year or periodically) and work on a continuous basis, while the
latter are temporary and cease to exist on completion of the task assigned to them.

[Source: Chapter 23: Indian Polity: M Laxmikanth]

QUESTION 68.
Consider the following statements

Civilsdaily
Page 0
Email: hello@civilsdaily.com
Exam Title : TS01 polity test L1
Email : sandeepbiswas221@gmail.com
Contact : 8250416813

1. The executive power of state shall be vested in governor.

2. After the 42nd Constitutional Amendment Act, the ministerial advice has been made binding on the
governor.

Which of the statements given above is/are correct?

www.freeupscmaterials.org
a) 1 only
b) 2 only

g
c) Both 1 and 2

or
d) Neither 1 nor 2
Correct Answer: A

s.
Your Answer: A
Explanation

Statement 1 is correct.
al
ri
-The executive power of the state shall be vested in the governor and shall be exercised by him either
directly or through officers subordinate to him in accordance with this Constitution (Article 154).
te

-The constitutional position of the governor differs from that of the president in the following two respects:
ma

1. While the Constitution envisages the possibility of the governor acting at times in his discretion, no such
possibility has been envisaged for the President.
sc

2. After the 42nd Constitutional Amendment (1976), ministerial advice has been made binding on the
up

President, but no such provision has been made with respect to the governor.
ee

[Source: Chapter 26: Indian Polity: M Laxmikanth]

QUESTION 69.
fr

Consider the following statements

1. The ministers shall hold office during the pleasure of the Chief Minister.

2. The council of ministers shall be collectively responsible to the legislative assembly of the state.

Which of the statements given above is/are correct?

a) 1 only
b) 2 only
c) Both 1 and 2
d) Neither 1 nor 2

Civilsdaily
Page 0
Email: hello@civilsdaily.com
Exam Title : TS01 polity test L1
Email : sandeepbiswas221@gmail.com
Contact : 8250416813

Correct Answer: B
Your Answer: B
Explanation

-The ministers shall hold office during the pleasure of the Governor and not Chief Minister.

-The council of ministers shall be collectively responsible to the legislative assembly of the state.

www.freeupscmaterials.org
[Source: Chapter 28: Indian Polity: M Laxmikanth]

g
or
.

QUESTION 70.

s.
Consider the following statements about special provisions related to state of Jammu and Kashmir

al
1. Part IV and Part IVA are applicable to J and K with exceptions and conditions.
ri
2. The Special leave jurisdiction of Supreme Court and jurisdictions of Election Commission and CAG are
not applicable to J and K.
te

Which of the statements given above is/are correct?


ma

a) 1 only
b) 2 only
sc

c) Both 1 and 2
d) Neither 1 nor 2
up

Correct Answer: D
Your Answer: Unanswered
ee

Explanation

-The State of J & K has its own Constitution and is administered according to that Constitution. Hence, Part
fr

VI of the Constitution of India (dealing with state governments) is not applicable to this state. The very
definition of state under this part does not include the State of J&K.

-Part III (dealing with Fundamental Rights) is applicable to the state with some exceptions and conditions.
The Fundamental Right to Property is still guaranteed in the state.

-Part IV (dealing with Directive Principles of State Policy) and Part IVA (dealing with Fundamental Duties)
are not applicable to the state.

--The special leave jurisdiction of the Supreme Court and the jurisdictions of the Election Commission and
the comptroller and auditor general are applicable to the state.

Civilsdaily
Page 0
Email: hello@civilsdaily.com
Exam Title : TS01 polity test L1
Email : sandeepbiswas221@gmail.com
Contact : 8250416813

-The High Court of J&K can issue writs only for the enforcement of the fundamental rights and not for any
other purpose

[Source: Chapter 32: Indian Polity: M Laxmikanth]

QUESTION 71.
The 73rd Constitutional amendment act is not applicable to which of the following states

www.freeupscmaterials.org
1. Nagaland

g
or
2. Meghalaya

3. Odisha

s.
4. Himachal Pradesh

Select the correct code


al
ri
a) 1 and 2
b) 2 and 3
te

c) 1 and 3
d) 3 and 4
ma

Correct Answer: A
Your Answer: A
sc

Explanation
up

-The 73rd Constitutional amendment act does not apply to the states of Jammu and Kashmir, Nagaland,
Meghalaya and Mizoram and certain other areas. These areas include,
ee

(a) the scheduled areas and the tribal areas in the states;
fr

(b) the hill area of Manipur for which a district council exists; and

(c) Darjeeling district of West Bengal for which Darjeeling Gorkha Hill Council exists.

-However, the Parliament may extend the provisions of this Part to the scheduled areas subject to such
exceptions and modifications as it may specify. Under this provision, the Parliament has enacted the
Provisions of the Panchayats (Extension to the Scheduled Areas) Act, 1996 (PESA).

[Source: Chapter 34: Indian Polity: M Laxmikanth]

QUESTION 72.
Consider the following statements

Civilsdaily
Page 0
Email: hello@civilsdaily.com
Exam Title : TS01 polity test L1
Email : sandeepbiswas221@gmail.com
Contact : 8250416813

1. District planning committee aims to consolidate the plans prepared by the panchayats and municipalities.

2. The district planning committee shall submit its report to the metropolitan planning committee which shall
lay it before the state government.

Which of the statements given above is/are correct?

www.freeupscmaterials.org
a) 1 only
b) 2 only

g
c) Both 1 and 2

or
d) Neither 1 nor 2
Correct Answer: A

s.
Your Answer: A
Explanation
al
-Every state shall constitute at the district level, a district planning committee to consolidate the plans
prepared by panchayats and municipalities in the district, and to prepare a draft development plan for the
ri
district as a whole.
te

-The act lays down that four-fifths of the members of a district planning committee should be elected by the
elected members of the district panchayat and municipalities in the district from amongst themselves.
ma

-The chairperson of such committee shall forward the development plan to the state government. Hence,
sc

DPC forwards its plan directly to the state government rather than submitting to MPC.

[Source: Chapter 35: Indian Polity: M Laxmikanth]


up

QUESTION 73.
ee

Which of the following statements regarding the Fundamentals Duties as mentioned in the Constitution of
India is/are correct?
fr

1. They can be enforced through writ jurisdiction.

2. They are a part of the Constituion since its adoption.

3. They are applicable only to citizens of India.

Which of the statements given above is/are correct?

a) 1 and 2 only
b) 1 and 3 only
c) 3 only
d) 2 and 3 only

Civilsdaily
Page 0
Email: hello@civilsdaily.com
Exam Title : TS01 polity test L1
Email : sandeepbiswas221@gmail.com
Contact : 8250416813

Correct Answer: C
Your Answer: C
Explanation

Fundamental Duties can be enforced through law, but not through writ jurisdiction.They are applicable only
to citizens of India and do not extend to foreigners.

www.freeupscmaterials.org
QUESTION 74.
Consider the following statements about National Commission for Scheduled Castes:

g
or
1. It is a statutory body established by the Parliament.

s.
2. It was established in 2004 as a separate commission for Scheduled Castes.

3. The Commision's members are appointed by the Parlaiment.

Which of the statements given above is/are correct?


al
ri

a) 1 and 2 only
b) 1 and 3 only
te

c) 2 only
d) 2 and 3 only
ma

Correct Answer: C
Your Answer: D
sc

Explanation
up

-It is a constitutional body


ee

-The commision's members are appointed by the president

QUESTION 75.
fr

What was the essence of the 24th Amendment to the Constitution of India?

a) It reversed the judgement of Golaknath case and enabled Parliament to abridge or take away any of the
Fundamental Rights under Article 368.
b) It laid the doctrine of the basic structure of the Constitution.
c) It reversed the judgement of Kesavanand Bharti case and dropped the doctrine of the basic structure of
the Constitution.
d) It denied the Parliament's power to amend the Fundamental Rights.
Correct Answer: A
Your Answer: A

Civilsdaily
Page 0
Email: hello@civilsdaily.com
Exam Title : TS01 polity test L1
Email : sandeepbiswas221@gmail.com
Contact : 8250416813

Explanation

The 24th Amendment to the Constitution of India enables Parliament to dilute Fundamental Rights to the
Constitution. It also amended Article 368 to provide that Parliament has power to amend any provision to
the Constitution.

QUESTION 76.

www.freeupscmaterials.org
The 86th Amendment of the Indian constitution added 11th Fundamental Duty in the Article IV-A. The 11th
Fundamental Duty is related to

g
or
a) Safeguard public property
b) Education of children between age of 6 to 14 years
c) Preserve the rich heritage of our composite culture

s.
d) None of the Above
Correct Answer: B
Your Answer: B
al
ri
Explanation

11th fundamental duty says 'who is a parent or guardian , to provide opportunities for education to his child,
te

or as the case may be, ward between the age of six and fourteen years.'
ma

QUESTION 77.
Who was the head of the 'Steering Committee' of the Constituent Assembly?
sc

a) Dr. Rajendra Prasad


b) HC Mukherjee
up

c) JB Kriplani
d) Gopinath Bardoloi
ee

Correct Answer: A
Your Answer: A
fr

Explanation

Dr. Rajendra Prasad was the head of the 'Steering Committee.'

QUESTION 78.
In the Indian Constitution, the method of election of president was taken from which of the Constitution?

a) Irish Constitution
b) British Constitution
c) French Constitution
d) Australian Constitution

Civilsdaily
Page 0
Email: hello@civilsdaily.com
Exam Title : TS01 polity test L1
Email : sandeepbiswas221@gmail.com
Contact : 8250416813

Correct Answer: A
Your Answer: B
Explanation

Other than this, we have taken Directive Principles of State Policy and nomination of members to Rajya
Sabha from Irish Constition.

www.freeupscmaterials.org
QUESTION 79.
Which of the following Constitutional Amendment is known as Mini-Constitution due to the important and

g
large number of changes made by it in various parts of the Constitution?

or
a) 73rd Amendment Act

s.
b) 44th Amendment Act
c) 7th Amendment Act
d) 42nd Amendment Act
Correct Answer: D
al
ri
Your Answer: D
Explanation
te

The 42nd Amendment Act (1976) is known as Mini- Constitution due to the important and large number of
changes made by it in various parts of the Constitution. However, in the Kesavananda Bharati case1
ma

(1973), the Supreme Court ruled that the constituent power of Parliament under Article 368 does not
enable it to alter the basic structure of the Constitution.
sc

QUESTION 80.
up

Which of the following function was not performed by the Constituent Assembly?

a) It ratified the India's membership of the Commonwealth in May 1949.


ee

b) It adopted the national flag on July 22, 1947.


c) It adopted the national anthem on January 24, 1950.
fr

d) All of the Above functions were performed by Constituent assembly


Correct Answer: D
Your Answer: Unanswered
Explanation

In addition to the making of the Constitution and enacting of ordinary laws, the Constituent Assembly also
performed the following functions:

1. It ratified the India's membership of the Commonwealth in May 1949.

2. It adopted the national flag on July 22, 1947.

Civilsdaily
Page 0
Email: hello@civilsdaily.com
Exam Title : TS01 polity test L1
Email : sandeepbiswas221@gmail.com
Contact : 8250416813

3. It adopted the national anthem on January 24, 1950.

4. It adopted the national song on January 24, 1950.

5. It elected Dr Rajendra Prasad as the first President of India on January 24, 1950.

QUESTION 81.

www.freeupscmaterials.org
Rules of Procedure' Committee of the Constituent Assembly was headed by

g
a) Dr. Rajendra Prasad
b) Sardar Patel

or
c) Jawahar Lal Nehru
d) JB Kriplani

s.
Correct Answer: A
Your Answer: A
Explanation
al
ri
Dr. Rajendra Prasad was the head of the 'Rules of Procedure Committee' in the Constituent Assembly
te

QUESTION 82.
Which of the following statements regarding the Preventive Detention is incorrect?
ma

a) If a person is arrested or detained under a law providing for preventive detention, then the protection
against arrest and detention under Article 22 (1) and 22 (2) shall not be available
sc

b) A detainee under preventive detention can have no right of personal liberty guaranteed by Article 19 or
Article 21
up

c) a person may be taken to preventive custody only for 3 months at the first instance
d) All are correct
ee

Correct Answer: D
Your Answer: D
fr

Explanation

The Article 22 (3) of the Indian constitution provides that, if a person is arrested or detained under a law
providing for preventive detention, then the protection against arrest and detention under Article 22 (1) and
22 (2) shall not be available.

Preventive detention on the other hand is action taken beforehand to prevent possible commitment of
crime. Preventive detention thus is action taken on grounds of suspicion that some wrong actions may be
done by the person concerned.

QUESTION 83.

Civilsdaily
Page 0
Email: hello@civilsdaily.com
Exam Title : TS01 polity test L1
Email : sandeepbiswas221@gmail.com
Contact : 8250416813

Consider the following statements w.r.t appointment of governor

1. A governor cannot be reappointed in the same state or any other state

2. Prime Minister can transfer a governor appointed to one state to another state for the rest of the term

Which of the above is/are correct?

www.freeupscmaterials.org
a) 1 only

g
b) 2 only

or
c) Both 1 and 2
d) Neither 1 nor 2
Correct Answer: D

s.
Your Answer: A
Explanation al
The President may transfer a Governor appointed to one state to another state for the rest of the term. A
ri
Governor whose term has expired may be reappointed in the same state or any other state.
te

QUESTION 84.
Consider the following statements w.r.t discretion of governor:
ma

1. Appointment of chief minister when no party has a clear-cut majority in the state legislative assembly
sc

2. Dissolution of the state legislative assembly if the council of ministers has lost its majority
up

3. Dismissal of the council of ministers when it cannot prove the confidence of the state legislative
assembly
ee

Governor has situational discretion in which of the above cases?


fr

a) 3 only
b) 2 only
c) 1 and 2 only
d) All of the above
Correct Answer: D
Your Answer: D
Explanation

The governor, like the president, also has situational discretion (i.e., the hidden discretion derived from the
exigencies of a prevailing political situation) in the following cases:

Civilsdaily
Page 0
Email: hello@civilsdaily.com
Exam Title : TS01 polity test L1
Email : sandeepbiswas221@gmail.com
Contact : 8250416813

1. Appointment of chief minister when no party has a clear-cut majority in the state legislative assembly or
when the chief minister in office dies suddenly and there is no obvious successor.

2. Dismissal of the council of ministers when it cannot prove the confidence of the state legislative
assembly.

3. Dissolution of the state legislative assembly if the council of ministers has lost its majority.

www.freeupscmaterials.org
QUESTION 85.

g
Consider the following statements regarding the Chief Minister

or
1. The constitution does not contain any specific procedure for the selection and appointment of the chief
minister.

s.
2. The constitution does not require that a person must prove his majority in the Legislative assembly
before he is appointed as the chief minister. al
ri
3. A person who is not a member of the state legislature cannot be appointed as chief minister.

Which of the statements are correct?


te

a) 1, 2 and 3
ma

b) 1 and 3
c) 2 and 3
sc

d) 1 and 2
Correct Answer: D
up

Your Answer: B
Explanation
ee

-1 and 2 are correct.


fr

-A person who is not a member of the state legislature can be appointed as chief minister for six months
maximum.

QUESTION 86.
Consider the following statements w.r.t powers of governor and president:

1. Ministerial advice is binding on both President and Governor

2. When PM or any Minister loses the 'No Confidence Motion', it is the President's discretion to dissolve the
Lower house or to look in to the alternative government farming while life of Lok Sabha is intact.

Civilsdaily
Page 0
Email: hello@civilsdaily.com
Exam Title : TS01 polity test L1
Email : sandeepbiswas221@gmail.com
Contact : 8250416813

Which of the above is/are correct?

a) 1 only
b) 2 only
c) Both 1 and 2
d) Neither 1 nor 2
Correct Answer: B

www.freeupscmaterials.org
Your Answer: D

g
Explanation

or
The Constitution envisages the possibility of the governor acting at times in his discretion, no such
possibility has been envisaged for the President. After the 42nd Constitutional Amendment (1976),

s.
ministerial advice has been made binding on the President, but no such provision has been made with
respect to the governor.

QUESTION 87.
al
ri
Consider the following statements regarding governor:

1. No Civil or criminal proceedings can be instituted against governor during his term of office
te

2. The oath of office to the governor is administered by the President


ma

Which of the above is/are correct?


sc

a) 1 only
b) 2 only
up

c) Both 1 and 2
d) Neither 1 nor 2
ee

Correct Answer: D
Your Answer: A
fr

Explanation

During his term of office, Governor is immune from any criminal proceedings, even in respect of his
personal acts. He cannot be arrested or imprisoned. However, after giving two months notice, civil
proceedings can be instituted against him during his term of office in respect of his personal acts.

The oath of office to the governor is administered by the chief justice of the concerned state high court and
in his absence, the senior-most judge of that court available.

QUESTION 88.
Consider the following statements regarding salary of governor:

Civilsdaily
Page 0
Email: hello@civilsdaily.com
Exam Title : TS01 polity test L1
Email : sandeepbiswas221@gmail.com
Contact : 8250416813

1. When the same person is appointed as the governor of two or more states, the emoluments and
allowances payable to him are shared by the states in such proportion as determined by the Parliament.

2. He is entitled to such emoluments, allowances and privileges as may be determined by Parliament.

Which of the above is/are correct?

www.freeupscmaterials.org
a) 1 only
b) 2 only

g
c) Both 1 and 2

or
d) Neither 1 nor 2
Correct Answer: B

s.
Your Answer: Unanswered
Explanation
al
He is entitled to such emoluments, allowances and privileges as may be determined by Parliament. When
the same person is appointed as the governor of two or more states, the emoluments and allowances
ri
payable to him are shared by the states in such proportion as determined by the PRESIDENT.
te

QUESTION 89.
Consider the following statements regarding appointment of governor:
ma

1. Governor is indirectly elected by a specially constituted electoral college of state legislative assembly
sc

2. Office of governor of a state is an employment under the Central government


up

Which of the above is/are correct?

a) 1 only
ee

b) 2 only
c) Both 1 and 2
fr

d) Neither 1 nor 2
Correct Answer: D
Your Answer: D
Explanation

The governor is neither directly elected by the people nor indirectly elected by a specially constituted
electoral college as is the case with the president. He is appointed by the president by warrant under his
hand and seal.

Civilsdaily
Page 0
Email: hello@civilsdaily.com
Exam Title : TS01 polity test L1
Email : sandeepbiswas221@gmail.com
Contact : 8250416813

As held by the Supreme Court in 1979, the office of governor of a state is not an employment under the
Central government. It is an independent constitutional office and is not under the control of or subordinate
to the Central government.

QUESTION 90.
Consider the following statements:

www.freeupscmaterials.org
1. The Governor acts as an agent of the central government.

g
2. Same person can be appointed as a governor for two or more states.

or
Which of the above is/are correct?

s.
a) 1 only
b) 2 only
c) Both 1 and 2
d) Neither 1 nor 2
al
ri
Correct Answer: C
Your Answer: C
te

Explanation
ma

Like the president, governor is a nominal executive head (titular or constitutional head). The governor also
acts as an agent of the central government. Therefore, the office of governor has a dual role. Usually, there
is a governor for each state, but the 7th Constitutional Amendment Act of 1956 facilitated the appointment
sc

of the same person as a governor for two or more states.


up

QUESTION 91.
Which of the following statements is/are not correct?
ee

1. The disputes regarding the election of President are enquired into by High Court and then an appeal
may be filed in Supreme Court.
fr

2. The governor is deemed to have vacated his seat on the day he is declared a nominee for the post of
President.

3. The pension of the judges of the Supreme Court and High Court is charged on the Consolidated Fund of
India.

Select the correct code

a) 2 and 3
b) 1 and 2
c) 1 and 3
d) 1, 2 and 3

Civilsdaily
Page 0
Email: hello@civilsdaily.com
Exam Title : TS01 polity test L1
Email : sandeepbiswas221@gmail.com
Contact : 8250416813

Correct Answer: B
Your Answer: Unanswered
Explanation

- 1 and 2 are wrong.

-The disputes regarding the election of President and Vice President are enquired into by Supreme Court

www.freeupscmaterials.org
and not High Court.

g
-The governor is not deemed to have vacated his seat on the day he is declared a nominee for the post of

or
President. He is deemed to have vacated his seat only if he is selected for the post of President.

s.
-The pension of the judges of the Supreme Court and High Court is charged on the Consolidated Fund of
India. However, the salary of the judges of the Supreme Court is charged on Consolidated Fund of India
al
and that of High Court judges is charged on the Consolidated Fund of state

[Source: Chapter 25, 22 and 26: Indian Polity: M Laxmikanth]


ri

QUESTION 92.
te

Consider the following sources of the Constitution and the features borrowed from them.

1. Canadian Constitution: Parliamentary privileges and bicameralism


ma

2. Australian Constitution: Concurrent List and joint sitting of Parliament


sc

3. Weimar Constitution of Germany: Fundamental duties


up

4. Irish Constitution: Directive Principles of State Policy


ee

Which of the following given above are correctly matched?


fr

a) 1, 2 and 4 only
b) 2 and 3 only
c) 1, 3 and 4 only
d) 2 and 4 only
Correct Answer: D
Your Answer: C
Explanation

-The Parliamentary privileges and bicameralism were borrowed from British constitution.

Civilsdaily
Page 0
Email: hello@civilsdaily.com
Exam Title : TS01 polity test L1
Email : sandeepbiswas221@gmail.com
Contact : 8250416813

-The Fundamental duties were borrowed from Soviet Constitution. The suspension of fundamental rights
during emergency was borrowed from Weimar Constitution of Germany.

QUESTION 93.
The second schedule of the constitution doesnt contain provision as to who among the following?

a) President

www.freeupscmaterials.org
b) Speaker of the House of people
c) CAG

g
d) The Chairman, UPSC

or
Correct Answer: D
Your Answer: D

s.
Explanation

al
-Second Schedule Provisions relating to the emoluments, allowances, privileges and so on of:

1. The President of India


ri

2. The Governors of States


te

3. The Speaker and the Deputy Speaker of the Lok Sabha


ma

4. The Chairman and the Deputy Chairman of the RajyaSabha


sc

5. The Speaker and the Deputy Speaker of the Legislative Assembly in the states
up

6. The Chairman and the Deputy Chairman of the Legislative Council in the states

7. The Judges of the Supreme Court


ee

8. The Judges of the High Courts


fr

9. The Comptroller and Auditor-General of India

[Source: Chapter 3: Indian Polity: M Laxmikanth]

QUESTION 94.
Indian federal system is based on:

a) Canadian model
b) American model
c) French model
d) Irish model

Civilsdaily
Page 0
Email: hello@civilsdaily.com
Exam Title : TS01 polity test L1
Email : sandeepbiswas221@gmail.com
Contact : 8250416813

Correct Answer: A
Your Answer: A
Explanation
QUESTION 95.
The difference between Articles 358 and 359 is given below:

1. Article 358 is confined to Fundamental Rights under the Article 19 only, whereas Article 359 extends to

www.freeupscmaterials.org
all those Fundamental Rights whose enforcement is suspended by President.

g
2. Article 358 operates only in case of external emergency, whereas Article 359 operated in case of both

or
external emergency as well as internal emergency.

s.
3. Article 358 extends to the entire country whereas Article 359 may extend to the entire country or a part of
it.

Which of the statements given above is/are incorrect?


al
ri
a) 1 and 2 only
b) 3 only
te

c) 1,2 and 3
d) None of the above
ma

Correct Answer: D
Your Answer: Unanswered
sc

Explanation
up

Article 358 is confined to Fundamental Rights under the Article 19 only, whereas Article 359 extends to all
those Fundamental Rights whose enforcement is suspended by the Presidential order.
ee

Article 358 operates only in case of external emergency, whereas Article 359 operated in case of both
external emergency as well as internal emergency.
fr

Article 358 extends to the entire country whereas Article 359 may extend to the entire country or a part of it.

QUESTION 96.
Right to life and personal liberty under Article 21 has been interpreted liberally and expanded, consider the
following statemenst in this regard:

1. The right against custodial death

2. Right to travel abroad

3. Right to primary education

Civilsdaily
Page 0
Email: hello@civilsdaily.com
Exam Title : TS01 polity test L1
Email : sandeepbiswas221@gmail.com
Contact : 8250416813

4. The right against public hanging

Which of the rights given above are covered under Article 21?

a) 1 and 3 only
b) 2 and 3 only
c) 2, 3 and 4 only

www.freeupscmaterials.org
d) 1,2,3 and 4
Correct Answer: D

g
Your Answer: C

or
Explanation

s.
The right against custodial death, right to travel abroad, right to primary education and the right against
public hanging are all covered under Article 21.

QUESTION 97.
al
Consider the following statements about the Attorney General of India:
ri

1.He is the Chief Law officer of the GoI.


te

2 His rank in the order of precedence is equal to central Cabinet Ministers


ma

3. He has the right to speak and take part in the proceedings of joint sitting of Parliament
sc

Which of the statements given above is/are correct?


up

a) 1 and 2 only
b) 1 and 3 only
c) 3 only
ee

d) 2 and 3
Correct Answer: B
fr

Your Answer: B
Explanation

Statement 2 is wrong. The cabinet Minister of union has rank 7 in the order of precednce while attorney
General is ranked 11.

QUESTION 98.
Consider the following statements:

1. The Comptroller and Auditor General of India can be removed by an address from both the Houses of
the Parliament only.

Civilsdaily
Page 0
Email: hello@civilsdaily.com
Exam Title : TS01 polity test L1
Email : sandeepbiswas221@gmail.com
Contact : 8250416813

2. The CAG of India has no control over the issue of money from the consolidated fund of India.

Which of the following options is correct?

a) 1 only
b) 2 only
c) 1 and 2 both

www.freeupscmaterials.org
d) Neither 1 nor 2
Correct Answer: C

g
Your Answer: Unanswered

or
Explanation

s.
The Comptroller and Auditor General of India can be removed by an address from both the Houses of the
Parliament only. The CAG of India has no control over the issue of money from the consolidated fund of
India. al
QUESTION 99.
ri
Consider the following statements:
te

1. The Constituent Assembly started functioning in December 1946.


2. Jawaharlal Nehru was the first President of the Constituent Assembly.
ma

3. The Constituent Assembly adopted the Constitution of India on 26th November 1949.
Which of the statements given above are correct?
sc

a) 1 and 2 only
b) 1 and 3 only
up

c) 3 only
d) 2 and 3 only
ee

Correct Answer: B
Your Answer: B
fr

Explanation

The Consituent Assembly held its first meeting on 9th December 1946 under the Presidentship of
Sachindand Sinha. In this meeting Rajendra Prasad was elected the President of Constituent Assembly for
the further sittings to be held.

QUESTION 100.
Consider the following statements:
1. The 74th Constitution Amendment Act gave constituional status to the Gram Sabha.
2. Gram Sabha is a body consisting of all the persons registered in the electoral rolls relating to a village
comprised within the area of Panchayat at the village level.

Civilsdaily
Page 0
Email: hello@civilsdaily.com
Exam Title : TS01 polity test L1
Email : sandeepbiswas221@gmail.com
Contact : 8250416813

3. The super-intendence, direction and control of the preparation of the electoral rolls and conduct of
elections to the panchayat shall be vested in election commission of India.

Which of the following statements given above is/are correct?

a) 1 and 2 only
b) 2 only

www.freeupscmaterials.org
c) 3 only
d) 2 and 3 only

g
Correct Answer: B

or
Your Answer: B
Explanation

s.
-The 73rd Constitution Amendment Act gave constituional status to the Gram Sabha. Thus statement 1 is
incorrect al
-The super-intendence, direction and control of the preparation of the electoral rolls and conduct of
ri
elections to the panchayat shall be vested in state election commission and not Election commission of
India.
te
ma
sc
up
ee
fr

Civilsdaily
Page 0
Email: hello@civilsdaily.com

Вам также может понравиться